(29 intermediate revisions by 2 users not shown)
Line 1: Line 1:
= [[ECE PhD Qualifying Exams|ECE Ph.D. Qualifying Exam]] in "Communication, Networks, Signal, and Image Processing" (CS)  =
+
[[Category:ECE]]
 +
[[Category:QE]]
 +
[[Category:CNSIP]]
 +
[[Category:problem solving]]
 +
[[Category:image processing]]
  
= [[ECE-QE_CS5-2011|Question 5, August 2011]], Part 1 =
+
<center>
 +
<font size= 4>
 +
[[ECE_PhD_Qualifying_Exams|ECE Ph.D. Qualifying Exam]]
 +
</font size>
  
:[[ECE-QE_CS5-2011_solusion-1|Part 1]],[[ECE-QE CS5-2011 solusion-2|2]]]
+
<font size= 4>
 +
Communication, Networking, Signal and Image Processing (CS)
  
 +
Question 5: Image Processing
 +
</font size>
 +
 +
August 2011
 +
</center>
 +
----
 +
----
 +
=Part 1 =
 +
Jump to [[ECE-QE_CS5-2011_solusion-1|Part 1]],[[ECE-QE CS5-2011 solusion-2|2]]
 
----
 
----
  
&nbsp;<font color="#ff0000"><span style="font-size: 19px;"><math>\color{blue}\text{1. } \left( \text{25 pts} \right) \text{ Let X, Y, and Z be three jointly distributed random variables with joint pdf} f_{XYZ}\left ( x,y,z \right )= \frac{3z^{2}}{7\sqrt[]{2\pi}}e^{-zy} exp \left [ -\frac{1}{2}\left ( \frac{x-y}{z}\right )^{2} \right ] \cdot 1_{\left[0,\infty \right )}\left(y \right )\cdot1_{\left[1,2 \right]} \left ( z \right) </math></span></font>  
+
&nbsp;<font color="#ff0000"><span style="font-size: 19px;"><math>\color{blue}\text{Consider the following discrete space system with input } x(m,n) \text{ and output } y(m,n).
 +
</math></span></font>  
  
'''<math>\color{blue}\left( \text{a} \right) \text{ Find the joint probability density function } f_{YZ}(y,z).</math>'''<br>  
+
&nbsp; &nbsp; &nbsp; &nbsp; &nbsp; &nbsp; &nbsp; &nbsp; <math>\color{blue}
 +
y(m,n) = \sum_{k=-\infty}^{\infty}{\sum_{l=-\infty}^{\infty}{x(m-k,n-l)h(k,l)}}.
 +
</math><br>  
  
===== <math>\color{blue}\text{Solution 1:}</math>  =====
 
  
<math> f_{YZ}\left (y,z \right )=\int_{-\infty}^{+\infty}f_{XYZ}\left(x,y,z \right )dx </math>&nbsp;
+
<math>\color{blue}
 +
\text{For parts a) and b) let}
 +
</math><br>
 +
&nbsp; &nbsp; &nbsp; &nbsp; &nbsp; &nbsp; &nbsp; &nbsp; <math>\color{blue}
 +
h(m,n)=sinc(mT,nT)
 +
</math><br>
 +
<math>\color{blue}  
 +
\text{where } T\leq1.
 +
</math><br>
  
&nbsp; &nbsp; &nbsp; &nbsp; &nbsp;<math> =\frac{3z^{2}}{7\sqrt[]{2\pi}}e^{-zy}\int_{-\infty}^{+\infty}exp\left[-\frac{1}{2}\left(\frac{x-y}{z} \right )^{2} \right ]dx\cdot 1_{[0,\infty)}
 
\left(y \right )\cdot1_{\left [1,2 \right ]}\left(z \right )</math><br>
 
  
<math>\text{But}\int_{-\infty}^{+\infty}exp\left[-\frac{1}{2}\left(\frac{x-y}{z} \right )^{2} \right ]dx \text{looks like the Gaussian pdf, so} </math>  
+
<math>\color{blue}
 +
\text{For parts c), d), and e) let}
 +
</math><br>
 +
&nbsp; &nbsp; &nbsp; &nbsp; &nbsp; &nbsp; &nbsp; &nbsp; <math>\color{blue}
 +
h(m,n)=sinc\left( \frac{(n+m)T}{\sqrt[]{2}},\frac{(n-m)T}{\sqrt[]{2}} \right)
 +
</math><br>
 +
<math>\color{blue}
 +
\text{where } T\leq1.
 +
</math><br>
  
<math> =\frac{3z^{2}}{7\sqrt[]{2\pi}}e^{-zy}
+
 
\underset{\sqrt[]{2\pi}z}{\underbrace{\frac{7\sqrt[]{2\pi}z}{7\sqrt[]{2\pi}z}  \int_{-\infty}^{+\infty}exp\left[-\frac{1}{2}\left(\frac{x-y}{z} \right )^{2} \right ]dx}}\cdot 1_{[0,\infty)}
+
 
\left(y \right )\cdot1_{\left [1,2 \right ]}\left(z \right )
+
<math>\color{blue}\text{a) Calculate the frequency response, }H \left( e^{j\mu},e^{j\nu} \right).</math><br>
 +
 
 +
===== <math>\color{blue}\text{Solution 1:}</math>  =====
 +
<math>\color{green}
 +
\text{Recall should be added:}
 +
</math>
 +
 
 +
<math>\color{green}
 +
f(am,bn) \overset{DTFT}{\Leftrightarrow } \frac{1}{|a||b|}F(\frac{\mu}{|a|},\frac{\nu}{|b|})
 +
</math>
 +
 
 +
<math>\color{green}
 +
sinc(m,n) \overset{DTFT}{\Leftrightarrow } rect(\mu,\nu)
 
</math>
 
</math>
  
 
<math>
 
<math>
=\frac{3z^{2}}{7}e^{-zy}\cdot 1_{[0,\infty)}
+
H(e^{j\mu},e^{j\nu}) = \frac{1}{T^2} rect(\frac{\mu}{T},\frac{\nu}{T})
\left(y \right )\cdot1_{\left [1,2 \right ]}\left(z \right )
+
 
</math>
 
</math>
  
Line 34: Line 78:
 
<math>\color{blue}\text{Solution 2:}</math>  
 
<math>\color{blue}\text{Solution 2:}</math>  
  
here put sol.2
+
<math>
 +
sinc(m,n) \rightarrow rect(\mu)rect(\nu)
 +
</math>
 +
 
 +
 
 +
<math>
 +
\Rightarrow sinc(mT,nT) \rightarrow \frac{1}{T^2}rect(\frac{\mu}{T})rect(\frac{\nu}{T})
 +
</math>
 +
 
 +
 
 +
<font face="serif"><span style="font-size: 19px;"><math>
 +
= H(e^{j\mu},e^{j\nu})
 +
</math></span></font>
 +
 
 +
<math>\color{green}
 +
\text{Here, the student uses the Separability property of the sinc and rect functions.}
 +
</math>
 
----
 
----
  
<math>\color{blue}\left( \text{b} \right) \text{Find}
+
<math>\color{blue}\text{b) Sketch the frequency response for } |\mu| < 2\pi \text{ and } |\nu| < 2\pi \text{ when } T = \frac{1}{2}
f_{x}\left( x|y,z\right )
+
 
</math><br>  
 
</math><br>  
  
<math>\color{blue}\text{Solution 1:}</math>  
+
<math>\color{green}
 +
\text{Recall should be added:}
 +
</math>
 +
 
 +
<math>\color{green}
 +
rect(t) = \left\{\begin{matrix}
 +
1, for |t|\leq \frac{1}{2}
 +
\\
 +
0, otherwise
 +
\end{matrix}\right.
 +
</math>
 +
 
 +
 
 +
<math>{\color{green}
 +
\text{Here, the following descriptions should be clarified:}
 +
}</math>
 +
 
 +
<font face="serif"><span style="font-size: 19px;"><math>{\color{green}
 +
\text{Using the separability property for rect function, for } T = \frac{1}{2} { we have:}
 +
}</math></span></font>
 +
 
 +
<font face="serif"><span style="font-size: 19px;"><math>{\color{green}
 +
H(e^{j\mu},e^{j\nu}) = \frac{1}{T^2} rect(\frac{\mu}{T},\frac{\nu}{T})
 +
}</math></span></font>
 +
 
 +
<font face="serif"><span style="font-size: 19px;"><math>{\color{green}
 +
= 4 rect(2\mu)rect(2\nu)
 +
}</math></span></font>
  
<font color="#ff0000"><span style="font-size: 17px;">'''<font face="serif"></font><math>
 
= \frac{f_{XYZ}\left( x,y,z\right )}{f_{YZ}\left(y,z \right )}
 
</math>'''</span></font><font color="#ff0000"><span style="font-size: 17px;">
 
</span></font>
 
  
'''<font face="serif"><math>
+
[[Image:QE_11_CS5_1_b.png]]
= \frac{e^{-\frac{1}{2}\left(\frac{x-y}{z} \right )^{2}}}{\sqrt[]{2\pi}z}
+
</math>&nbsp;&nbsp;</font>'''
+
  
 +
<font face="serif"><span style="font-size: 19px;"><math>{\color{red}
 +
\text{In this sketch it is not mentioned that the gain is } 4.
 +
}</math></span></font>
 
----
 
----
  
 
<math>\color{blue}\text{Solution 2:}</math><br>  
 
<math>\color{blue}\text{Solution 2:}</math><br>  
  
sol2 here
+
<math>
 +
T = \frac{1}{2}, H(e^{j\mu},e^{j\nu}) = 4rect(2\mu)rect(2\nu)
 +
</math>
 +
 
 +
[[Image:QE_11_CS5_1_b_sol2.PNG]]
 +
 
 +
 
 
----
 
----
  
<math>\color{blue}\left( \text{c} \right) \text{Find}  
+
<math>\color{blue}\text{c) Calculate the frequency response, }H \left( e^{j\mu},e^{j\nu} \right).</math><br>  
f_{Z}\left( z\right )
+
</math><br>  
+
  
 
<math>\color{blue}\text{Solution 1:}</math>  
 
<math>\color{blue}\text{Solution 1:}</math>  
  
<font color="#ff0000"><span style="font-size: 17px;">'''<font face="serif"></font><math>
+
<math>\color{green}
=\int_{0}^{+\infty}{f_{YZ}\left(y,z \right )dy}
+
\text{Recall should be added:}
 +
</math>
 +
 
 +
<math>\color{green}
 +
f \left ( A \begin{bmatrix}
 +
m
 +
\\
 +
n
 +
\end{bmatrix} \right) \overset{DTFT}{\Leftrightarrow } \frac{1}{|A|^{-1}}F([\mu, \nu] A^{-1})
 +
</math>
 +
 
 +
 
 +
<font color="#ff0000"><span style="font-size: 17px;">'''<font face="serif"></font><math>\color{green}
 +
\text{ In this case, A}= \begin{bmatrix}
 +
\frac{1}{\sqrt{2}} &\frac{1}{\sqrt{2}} \\  
 +
-\frac{1}{\sqrt{2}} &\frac{1}{\sqrt{2}}
 +
\end{bmatrix} \text{, hence:}
 
</math>'''</span></font><font color="#ff0000"><span style="font-size: 17px;">
 
</math>'''</span></font><font color="#ff0000"><span style="font-size: 17px;">
 
</span></font>  
 
</span></font>  
  
'''<font face="serif"><math>
+
 
=\frac{3z^{2}}{7}\cdot1_{\left[1,2 \right ]}(z)
+
<font color="#ff0000"><span style="font-size: 17px;">'''<font face="serif"></font><math>
</math>&nbsp;&nbsp;</font>'''
+
H(e^{j\mu},e^{j\nu}) = \frac{1}{T^2} rect \left ( \frac{(\mu + \nu)}{\sqrt{2}T},\frac{(\nu - \mu)}{\sqrt{2}T} \right )
 +
</math>'''</span></font><font color="#ff0000"><span style="font-size: 17px;">
 +
</span></font>  
  
 
----
 
----
Line 78: Line 183:
 
<math>\color{blue}\text{Solution 2:}</math><br>  
 
<math>\color{blue}\text{Solution 2:}</math><br>  
  
sol2 here
+
<math>
 +
\left ( \frac{(n + m)T}{\sqrt{2}},\frac{(n - m)T}{\sqrt{2}} \right) = \begin{bmatrix}
 +
\frac{1}{\sqrt{2}} &\frac{1}{\sqrt{2}} \\
 +
-\frac{1}{\sqrt{2}} &\frac{1}{\sqrt{2}}
 +
\end{bmatrix} \cdot \begin{pmatrix}
 +
mT\\
 +
nT
 +
\end{pmatrix} = A \cdot \begin{pmatrix}
 +
mT\\
 +
nT
 +
\end{pmatrix}
 +
</math>
 +
 
 +
<math>
 +
\text{As } |A| = 1, A^{-1} = A^T, sinc \left( A \begin{pmatrix}
 +
mT\\
 +
nT
 +
\end{pmatrix} \right)
 +
 
 +
\overset{\mathcal{F}}{\rightarrow} F \left( A \begin{pmatrix}
 +
\mu\\
 +
\nu
 +
\end{pmatrix} \right)
 +
 
 +
 
 +
</math>
 +
 
 +
<math>
 +
= \frac{1}{T^2} rect \left ( \frac{(\mu + \nu)}{\sqrt{2}T},\frac{(\nu - \mu)}{\sqrt{2}T} \right )
 +
</math>
 +
 
 
----
 
----
  
<math>\color{blue}\left( \text{d} \right) \text{Find}
+
<math>\color{blue}\text{d) Sketch the frequency response for } |\mu| < 2\pi \text{ and } |\nu| < 2\pi \text{ when } T = \frac{1}{2}
f_{Y}\left(y|z \right )
+
 
</math><br>  
 
</math><br>  
  
 
<math>\color{blue}\text{Solution 1:}</math>  
 
<math>\color{blue}\text{Solution 1:}</math>  
  
<font color="#ff0000"><span style="font-size: 17px;">'''<font face="serif"></font><math>
+
<math>\color{green}
=\frac{f_{YZ}\left(y,z \right )}{f_{Z}(z)}</math>'''</span></font><font color="#ff0000"><span style="font-size: 17px;">
+
\text{Recall should be added: Since A is an orthogonal matrix, this transformation is rotationally invariant.}
</span></font>  
+
</math>
  
'''<font face="serif"><math>
+
<font face="serif"><span style="font-size: 19px;"><math>\color{green}
=e^{-zy}z\cdot1_{\left[0,\infty \right )}(y)
+
H(e^{j\mu},e^{j\nu}) = \frac{1}{T^2} rect \left ( \frac{(\mu + \nu)}{\sqrt{2}T},\frac{(\nu - \mu)}{\sqrt{2}T} \right )
</math>&nbsp;&nbsp;</font>'''
+
</math></span></font>
 +
 
 +
<font face="serif"><span style="font-size: 19px;"><math>\color{green}
 +
= 4 rect \left (\sqrt{2} (\mu + \nu),\sqrt{2}(\nu - \mu) \right )
 +
</math></span></font>
 +
 
 +
<font face="serif"><span style="font-size: 19px;"><math>\color{green}
 +
\text{Or}
 +
</math></span></font>
 +
 
 +
<font face="serif"><span style="font-size: 19px;"><math>\color{green}
 +
= 4 rect \left (\sqrt{2} (\mu + \nu) \right) rect \left (\sqrt{2}(\nu - \mu) \right )
 +
</math></span></font>
 +
 
 +
 
 +
[[Image:QE_11_CS5_1_d.PNG]]
 +
 
 +
 
 +
<font face="serif"><span style="font-size: 19px;"><math>{ \color{red}
 +
\text{This sketch is partially correct: The cut-offs should be divided by } 4!
 +
}</math></span></font>
 +
 
 +
<font face="serif"><span style="font-size: 19px;"><math>{ \color{red}
 +
\text{ Also, it should be mentioned that the gain is} 4.
 +
}</math></span></font>
  
 
----
 
----
Line 99: Line 257:
 
<math>\color{blue}\text{Solution 2:}</math><br>  
 
<math>\color{blue}\text{Solution 2:}</math><br>  
  
sol2 here
+
<math>
 +
T = \frac{1}{2}, H(e^{j\mu},e^{j\nu}) = 4rect(\sqrt{2}(\mu + \nu))rect(\sqrt{2}(\nu - \mu))
 +
</math>
 +
 
 +
[[Image:QE_11_CS5_1_d_sol2.PNG]]
 +
 
 
----
 
----
<math>\color{blue}\left( \text{e} \right) \text{Find}  
+
<math>\color{blue}\text{e) Calculate } y(m,n) \text{ when } x(m,n)=1.</math><br>
f_{XY}\left(x,y|z \right )
+
 
</math><br>  
+
  
 
<math>\color{blue}\text{Solution 1:}</math>  
 
<math>\color{blue}\text{Solution 1:}</math>  
  
<font color="#ff0000"><span style="font-size: 17px;">'''<font face="serif"></font><math>
 
=\frac{f_{XYZ}\left(x,y,z \right )}{f_{Z}(z)}
 
</math>'''</span></font><font color="#ff0000"><span style="font-size: 17px;">
 
</span></font>
 
  
'''<font face="serif"><math>
+
<math>
=\frac{e^{-zy}}{\sqrt[]{2\pi}}e^{-\frac{1}{2}\left(\frac{x-y}{z} \right )^{2}}\cdot1_{\left[0,\infty \right )}(y)
+
Y(e^{j\mu},e^{j\nu}) = \delta(e^{j\mu},e^{j\nu}) \cdot H(e^{j\mu},e^{j\nu})
</math>&nbsp;&nbsp;</font>'''
+
</math>
 +
 
 +
<math>
 +
= \frac{1}{T^2} rect (0,0) = 4
 +
</math>
 +
 
 +
<math>
 +
\Rightarrow y(m,n) = 4\delta(m,n)
 +
</math>
  
 
----
 
----
Line 120: Line 286:
 
<math>\color{blue}\text{Solution 2:}</math><br>  
 
<math>\color{blue}\text{Solution 2:}</math><br>  
  
sol2 here
+
<math>
 +
y(m,n) = x(m,n) \cdot H(e^{j0},e^{j0}) = 4
 +
</math>
 +
 
 +
<math>\color{red}
 +
\text{The final answer is correct, but the student has skipped some parts of the derivation and the notations do not sound right.}
 +
</math>
 +
 
 
----
 
----
  
Line 132: Line 305:
 
----
 
----
  
[[ECE PhD Qualifying Exams|Back to ECE Qualifying Exams (QE) page]]
+
[[ECE PhD Qualifying Exams|Back to ECE Qualifying Exams (QE) page]]
 
+
[[Category:ECE]] [[Category:QE]] [[Category:CS]] [[Category:Problem_solving]]
+

Latest revision as of 10:31, 13 September 2013


ECE Ph.D. Qualifying Exam

Communication, Networking, Signal and Image Processing (CS)

Question 5: Image Processing

August 2011



Part 1

Jump to Part 1,2


 $ \color{blue}\text{Consider the following discrete space system with input } x(m,n) \text{ and output } y(m,n). $

                $ \color{blue} y(m,n) = \sum_{k=-\infty}^{\infty}{\sum_{l=-\infty}^{\infty}{x(m-k,n-l)h(k,l)}}. $


$ \color{blue} \text{For parts a) and b) let} $
                $ \color{blue} h(m,n)=sinc(mT,nT) $
$ \color{blue} \text{where } T\leq1. $


$ \color{blue} \text{For parts c), d), and e) let} $
                $ \color{blue} h(m,n)=sinc\left( \frac{(n+m)T}{\sqrt[]{2}},\frac{(n-m)T}{\sqrt[]{2}} \right) $
$ \color{blue} \text{where } T\leq1. $


$ \color{blue}\text{a) Calculate the frequency response, }H \left( e^{j\mu},e^{j\nu} \right). $

$ \color{blue}\text{Solution 1:} $

$ \color{green} \text{Recall should be added:} $

$ \color{green} f(am,bn) \overset{DTFT}{\Leftrightarrow } \frac{1}{|a||b|}F(\frac{\mu}{|a|},\frac{\nu}{|b|}) $

$ \color{green} sinc(m,n) \overset{DTFT}{\Leftrightarrow } rect(\mu,\nu) $

$ H(e^{j\mu},e^{j\nu}) = \frac{1}{T^2} rect(\frac{\mu}{T},\frac{\nu}{T}) $


$ \color{blue}\text{Solution 2:} $

$ sinc(m,n) \rightarrow rect(\mu)rect(\nu) $


$ \Rightarrow sinc(mT,nT) \rightarrow \frac{1}{T^2}rect(\frac{\mu}{T})rect(\frac{\nu}{T}) $


$ = H(e^{j\mu},e^{j\nu}) $

$ \color{green} \text{Here, the student uses the Separability property of the sinc and rect functions.} $


$ \color{blue}\text{b) Sketch the frequency response for } |\mu| < 2\pi \text{ and } |\nu| < 2\pi \text{ when } T = \frac{1}{2} $

$ \color{green} \text{Recall should be added:} $

$ \color{green} rect(t) = \left\{\begin{matrix} 1, for |t|\leq \frac{1}{2} \\ 0, otherwise \end{matrix}\right. $


$ {\color{green} \text{Here, the following descriptions should be clarified:} } $

$ {\color{green} \text{Using the separability property for rect function, for } T = \frac{1}{2} { we have:} } $

$ {\color{green} H(e^{j\mu},e^{j\nu}) = \frac{1}{T^2} rect(\frac{\mu}{T},\frac{\nu}{T}) } $

$ {\color{green} = 4 rect(2\mu)rect(2\nu) } $


QE 11 CS5 1 b.png

$ {\color{red} \text{In this sketch it is not mentioned that the gain is } 4. } $


$ \color{blue}\text{Solution 2:} $

$ T = \frac{1}{2}, H(e^{j\mu},e^{j\nu}) = 4rect(2\mu)rect(2\nu) $

QE 11 CS5 1 b sol2.PNG



$ \color{blue}\text{c) Calculate the frequency response, }H \left( e^{j\mu},e^{j\nu} \right). $

$ \color{blue}\text{Solution 1:} $

$ \color{green} \text{Recall should be added:} $

$ \color{green} f \left ( A \begin{bmatrix} m \\ n \end{bmatrix} \right) \overset{DTFT}{\Leftrightarrow } \frac{1}{|A|^{-1}}F([\mu, \nu] A^{-1}) $


$ \color{green} \text{ In this case, A}= \begin{bmatrix} \frac{1}{\sqrt{2}} &\frac{1}{\sqrt{2}} \\ -\frac{1}{\sqrt{2}} &\frac{1}{\sqrt{2}} \end{bmatrix} \text{, hence:} $


$ H(e^{j\mu},e^{j\nu}) = \frac{1}{T^2} rect \left ( \frac{(\mu + \nu)}{\sqrt{2}T},\frac{(\nu - \mu)}{\sqrt{2}T} \right ) $


$ \color{blue}\text{Solution 2:} $

$ \left ( \frac{(n + m)T}{\sqrt{2}},\frac{(n - m)T}{\sqrt{2}} \right) = \begin{bmatrix} \frac{1}{\sqrt{2}} &\frac{1}{\sqrt{2}} \\ -\frac{1}{\sqrt{2}} &\frac{1}{\sqrt{2}} \end{bmatrix} \cdot \begin{pmatrix} mT\\ nT \end{pmatrix} = A \cdot \begin{pmatrix} mT\\ nT \end{pmatrix} $

$ \text{As } |A| = 1, A^{-1} = A^T, sinc \left( A \begin{pmatrix} mT\\ nT \end{pmatrix} \right) \overset{\mathcal{F}}{\rightarrow} F \left( A \begin{pmatrix} \mu\\ \nu \end{pmatrix} \right) $

$ = \frac{1}{T^2} rect \left ( \frac{(\mu + \nu)}{\sqrt{2}T},\frac{(\nu - \mu)}{\sqrt{2}T} \right ) $


$ \color{blue}\text{d) Sketch the frequency response for } |\mu| < 2\pi \text{ and } |\nu| < 2\pi \text{ when } T = \frac{1}{2} $

$ \color{blue}\text{Solution 1:} $

$ \color{green} \text{Recall should be added: Since A is an orthogonal matrix, this transformation is rotationally invariant.} $

$ \color{green} H(e^{j\mu},e^{j\nu}) = \frac{1}{T^2} rect \left ( \frac{(\mu + \nu)}{\sqrt{2}T},\frac{(\nu - \mu)}{\sqrt{2}T} \right ) $

$ \color{green} = 4 rect \left (\sqrt{2} (\mu + \nu),\sqrt{2}(\nu - \mu) \right ) $

$ \color{green} \text{Or} $

$ \color{green} = 4 rect \left (\sqrt{2} (\mu + \nu) \right) rect \left (\sqrt{2}(\nu - \mu) \right ) $


QE 11 CS5 1 d.PNG


$ { \color{red} \text{This sketch is partially correct: The cut-offs should be divided by } 4! } $

$ { \color{red} \text{ Also, it should be mentioned that the gain is} 4. } $


$ \color{blue}\text{Solution 2:} $

$ T = \frac{1}{2}, H(e^{j\mu},e^{j\nu}) = 4rect(\sqrt{2}(\mu + \nu))rect(\sqrt{2}(\nu - \mu)) $

QE 11 CS5 1 d sol2.PNG


$ \color{blue}\text{e) Calculate } y(m,n) \text{ when } x(m,n)=1. $


$ \color{blue}\text{Solution 1:} $


$ Y(e^{j\mu},e^{j\nu}) = \delta(e^{j\mu},e^{j\nu}) \cdot H(e^{j\mu},e^{j\nu}) $

$ = \frac{1}{T^2} rect (0,0) = 4 $

$ \Rightarrow y(m,n) = 4\delta(m,n) $


$ \color{blue}\text{Solution 2:} $

$ y(m,n) = x(m,n) \cdot H(e^{j0},e^{j0}) = 4 $

$ \color{red} \text{The final answer is correct, but the student has skipped some parts of the derivation and the notations do not sound right.} $


"Communication, Networks, Signal, and Image Processing" (CS)- Question 5, August 2011

Go to


Back to ECE Qualifying Exams (QE) page

Alumni Liaison

Correspondence Chess Grandmaster and Purdue Alumni

Prof. Dan Fleetwood